LSAT and Law School Admissions Forum

Get expert LSAT preparation and law school admissions advice from PowerScore Test Preparation.

User avatar
 Dave Killoran
PowerScore Staff
  • PowerScore Staff
  • Posts: 5852
  • Joined: Mar 25, 2011
|
#94662
Complete Question Explanation
(The complete setup for this game can be found here: lsat/viewtopic.php?f=280&t=8537)

The correct answer choice is (A)

Answer choice (A) cannot occur, and thus is the correct answer. Under answer choice (A), there would only be a total of four works available to be selected, a violation of the condition in the game scenario that requires at least five works to be selected.

The remaining answer choices all could be true. Note that answer choice (D) is proven possible by the hypothetical produced in question #7.
 afinelli
  • Posts: 26
  • Joined: Sep 05, 2011
|
#1803
On this game, I got stuck on #11. I didn't think any of the answer choices could be right because all the choices could be true. The correct answer choice, A, I thought could be true because four French plays and one Russian play could be selected, satisfying all the rules (I think?). What am I missing?
Thanks for any help.
User avatar
 Dave Killoran
PowerScore Staff
  • PowerScore Staff
  • Posts: 5852
  • Joined: Mar 25, 2011
|
#1808
The "four French plays and one Russian play" scenario you suggest won't work because the answer choice specifies that "exactly one play" is selected.

Although I think maybe you meant 4 French works and not 4 French plays, that still won't work numerically. Basically, the problem with (A) is that you can't get to a total of 5 works; (A) only allows for 4 works at most (3 French novels and 1 Russian or French play).

Does that make sense?
 afinelli
  • Posts: 26
  • Joined: Sep 05, 2011
|
#1818
Of course. Thanks!
User avatar
 Pandaa
  • Posts: 2
  • Joined: Apr 16, 2023
|
#105485
Relating to question #11, would this be correct for conditional rule #4?

Conditional:
Fn & Fn —> ~Rp

Contrapositive:
Rp —> ~Fn or ~Fn

If so, is this why would could select Fn Fn Fn, and because we select Rp we can’t select one of the Fp’s, due to the “or”? This is how I arrived at answer choice (A), because Fn Fn Fn Rn Rp would be true. If this thinking is wrong, I need help understanding why (A) is correct. I know why (B-E) is incorrect.
User avatar
 Jeff Wren
PowerScore Staff
  • PowerScore Staff
  • Posts: 388
  • Joined: Oct 19, 2022
|
#105500
Hi Pandaa,

I'm not completely sure that I understand your question, but I'll try to clarify.

First, the fourth rule of the game states that if both French plays are selected, then the Russian play is not selected.

From your diagram, it looks like you have the rule saying if both French novels are selected, which is incorrect.

Second, the reason that Answer A cannot work (and is therefore the correct answer to question 11) has nothing to do with the fourth rule.

The game scenario states that at least five works must be selected.

In Answer A, no Russian novels and exactly (meaning only) 1 play is selected. This cannot work because if no Russian novels are selected, that only leaves the 3 French novels and the 1 play to be selected. This only adds up to 4 items, but the game requires at least 5 selected.

Get the most out of your LSAT Prep Plus subscription.

Analyze and track your performance with our Testing and Analytics Package.